Power series: Why is this power series equal to log(2)?

Click For Summary
SUMMARY

The power series ##\sum_{n=0}^\infty (-1)^n \frac {x^{n+1}}{n+1}## converges to ##\log(2)## when evaluated at ##x=1##. The discussion highlights the use of the Leibniz test for convergence but emphasizes the importance of integrating the series for ##\frac{1}{1+x}##, which leads to the conclusion that the series represents ##\log(1+x)##. The realization that substituting ##x=1## results in the convergence to ##\log(2)## is a key insight shared among participants.

PREREQUISITES
  • Understanding of power series and their convergence
  • Familiarity with the Leibniz test for alternating series
  • Knowledge of logarithmic functions and their properties
  • Basic calculus concepts, particularly integration techniques
NEXT STEPS
  • Study the derivation of the Taylor series for ##\log(1+x)##
  • Explore the application of the Leibniz test in various series
  • Learn about the properties of alternating series and their convergence criteria
  • Investigate integration techniques for series and their applications in calculus
USEFUL FOR

Mathematicians, calculus students, and anyone interested in series convergence and logarithmic functions will benefit from this discussion.

DottZakapa
Messages
239
Reaction score
17
Homework Statement
##\sum_{n=0}^\infty (-1)^n \frac {x^\left(n+1\right)}{n+1}## for x=1
Relevant Equations
power series
##\sum_{n=0}^\infty (-1)^n \frac {x^\left(n+1\right)}{n+1}## for x=1

##\sum_{n=0}^\infty (-1)^n \frac {1^\left(n+1\right)}{n+1}##

i've tried leibniz test but i can only find that it converges

why is this power equal to ##log(2)##?

i've also tried with ##\sum_{n=0}^\infty\log \left (1+\frac 1 {n+1}\right)##

but does not lead to that result

what am i missing ?
 
Last edited by a moderator:
Physics news on Phys.org
Hint: Try integrating the series for ##\frac{1}{1+x}##.
 
vela said:
Hint: Try integrating the series for ##\frac{1}{1+x}##.

which is log(1+x)
:doh:
ok, didn't understood that if it converges i could plug there the value of x to see at which value converges. Thank you. :smile:
 
Question: A clock's minute hand has length 4 and its hour hand has length 3. What is the distance between the tips at the moment when it is increasing most rapidly?(Putnam Exam Question) Answer: Making assumption that both the hands moves at constant angular velocities, the answer is ## \sqrt{7} .## But don't you think this assumption is somewhat doubtful and wrong?

Similar threads

  • · Replies 2 ·
Replies
2
Views
2K
  • · Replies 3 ·
Replies
3
Views
1K
  • · Replies 7 ·
Replies
7
Views
2K
  • · Replies 2 ·
Replies
2
Views
2K
  • · Replies 1 ·
Replies
1
Views
2K
  • · Replies 3 ·
Replies
3
Views
2K
Replies
8
Views
3K
  • · Replies 17 ·
Replies
17
Views
2K
  • · Replies 1 ·
Replies
1
Views
968
  • · Replies 5 ·
Replies
5
Views
2K